Euler's formula + Handshake Theorem

  • Thread starter Thread starter Natasha1
  • Start date Start date
  • Tags Tags
    Formula Theorem
Click For Summary
SUMMARY

This discussion focuses on demonstrating the non-planarity of a graph using Euler's formula and the Handshake Theorem. Euler's formula, defined as v + f = e + 2, was applied with v = 8 and e = 21, leading to an incorrect calculation of f = 15. Additionally, the Handshake Theorem, which states that 2e ≥ 3f, indicated that f must be less than or equal to 14, further confirming the graph's non-planarity. The contradiction in results from both theorems establishes that the graph cannot be planar.

PREREQUISITES
  • Understanding of Euler's formula in graph theory
  • Familiarity with the Handshake Theorem
  • Basic knowledge of graph properties (vertices, edges, faces)
  • Ability to perform algebraic manipulations
NEXT STEPS
  • Study advanced graph theory concepts, including Kuratowski's theorem
  • Explore planar graph testing algorithms
  • Learn about graph embeddings and their applications
  • Investigate the implications of non-planarity in network design
USEFUL FOR

Mathematicians, computer scientists, and students studying graph theory, particularly those interested in planar graphs and their properties.

Natasha1
Messages
494
Reaction score
9
Using Euler's formula and the Handshake Theorem, how can I show that this graph is non-planar? (see graph attached)


My answer:

Euler's formula states that v+f = e+2

Here

v=8
e=21
f=?

so 8+f=21+2
hence f=15 which is not true as there are many more. Hence the graph is non-planar.

Using the Handshake Theorem that states that 2e >= 3f
we get ( 2(21) )/ 3 >= f which gives f <= 14 which again is not true, hence the graph is non-planar.

Am I correct?
 

Attachments

  • Pic 11.jpg
    Pic 11.jpg
    16.2 KB · Views: 451
Last edited:
Physics news on Phys.org
Attatchment is still pending approval, but this is just a matter of counting right? I mean you know if it's planar e, v, and f satisfy some simple relations, therefore if they don't satisfy them it's not planar. Have some confidence!
 
rather than using the two theorems to get results, then say that they are not true, would it not be better to use the fact that they are contradictory?
 
Question: A clock's minute hand has length 4 and its hour hand has length 3. What is the distance between the tips at the moment when it is increasing most rapidly?(Putnam Exam Question) Answer: Making assumption that both the hands moves at constant angular velocities, the answer is ## \sqrt{7} .## But don't you think this assumption is somewhat doubtful and wrong?

Similar threads

Replies
22
Views
3K
  • · Replies 3 ·
Replies
3
Views
2K
Replies
6
Views
2K
  • · Replies 3 ·
Replies
3
Views
2K
Replies
1
Views
2K
Replies
10
Views
2K
Replies
10
Views
2K
  • · Replies 1 ·
Replies
1
Views
2K
  • · Replies 2 ·
Replies
2
Views
3K
  • · Replies 2 ·
Replies
2
Views
2K